Variation gegenüber RabcdRabcdR_{abcd}? Wie berechnet man∂R∂Rabcd=12(gacgbd−gadgbc)∂R∂Rabcd=12(gacgbd−gadgbc)\frac{\partial R}{\partial R_{abcd}}=\frac{1}{2}( g^{ac} g^{bd} - g^{ad} g^{bc})?

In letzter Zeit habe ich mich für Walds Entropieformel interessiert. In diesem Formalismus gibt es einen Tensor P A B C D = L R A B C D .

Unter einigen Papieren erwähnen sie P A B C D , die seine symmetrisch antisymmetrischen Eigenschaften beschreibt. Aber ich möchte die explizite Form von wissen P in bestimmten Fällen.

Es gibt bekannte Ergebnisse der Einstein-Hilbert-Aktion (sie erwähnen, P A B C D = 1 2 ( G A C G B D G A D G B C ) für den Fall Einstein Hilbert), also gehe ich davon aus

R R A B C D = 1 2 ( G A C G B D G A D G B C ) ( ? )
Wo G ist die übliche symmetrische Metrik, R A B C D ist der Riemann-Krümmungstensor, und R ist der Ricci-Skalar.

Es scheint, dass sie behandeln R A B C D Und G A B unabhängig, also ist mein erster Versuch zersetzen R = G A C G B D R A B C D , und versuchen Sie zu berechnen R P Q R S R A B C D , aus den symmetrischen Argumenten

R A B C D R P Q R S = δ A B P Q δ C D R S + δ C D P Q δ A B R S
Aber stecken Sie diese an R , erhalte ich eine etwas andere oben gezeigte Antwort.

Mache ich etwas falsch?

Wenn Sie diese Art von Ableitung erlebt haben, haben Sie irgendwelche Hinweise oder Ratschläge für diese Art von algebraischer Berechnung?

Hallo @phy_math: Ist das aus Walds GR-Buch? Welche genaue Referenz? Welche Seite?
@Qmechanic, es stammt aus Walds Artikel über BlackHoles. Ich sehe diese Art von Gleichungen oft wie modifizierte Schwerkraft, F ( R ) , und Gauss-Bonnet-Schwerkraft, höhere Ableitungstheorien usw.

Antworten (1)

Beachten Sie das zunächst

R = R A C G A C = R A B C D G A C G B D .
Die Antisymmetrien von R implizieren
R A B C D ( G A C G B D + G A D G B C ) = 0 , R = R A B C D ( ( 1 k ) G A C G B D k G A D G B C )
für jede Konstante k . Seit R R A B C D müssen beim Austausch antisymmetrisch sein A mit B oder C mit D , es ist der Koeffizient, der durch Auswahl erhalten wird k so dass 1 k = k , dh k = 1 2 . Somit
R R A B C D = 1 2 ( G A C G B D G A D G B C ) .
Beachten Sie das A B C D C D A B Symmetrie.

Danke!, Mir ist aufgefallen, dass symmetrisches Argument für die Variation von funktioniert R , aber wie wär's R A B R A B Fall?, Ich werde es als einen anderen Beitrag machen.
@phy_math Hinweis: Wenn X A B C D e F := R A B R C D e F Dann ( R A B R A B ) R C D e F = 2 R A B X A B C D e F .
Mit Ihrem Kommentar habe ich einen Beitrag zu physical.stackexchange.com/questions/301787/… gemacht , dennoch bin ich etwas verwirrend, wenn ich Begriffe für bestimmte Paare symmetrisiert und antisymmetrisiert mache.
Symmetrien: R X j = R j X , R X j v , w = R X j w , v , R X j z + R j z X + R z X j , R X j v , w = R v w X , j wobei alle Vektoren Elemente von sind T P M .